Processing math: 100%

Wednesday, 1 May 2019

Finite Difference, Discrete MacLaurin, dan Integer-Valued Polynomials

[Pelatihan Tim IMO 2019 Indonesia , Soto Bang Sawit]

Sebuah monomial M_k(z)=z^k dapat kita gunakan dalam expansi Maclaurin-Taylor dari fungsi f  sebagai  berikut:

f(z) = \sum_{k=0}^{\infty} a_k M_k(z)

dimana a_k = \frac{d^k (M_k(z))}{dz}\big\lvert_{z=0},  yakni merupakan turunan ke-k  dari M_k(z) yang dievaluasi di z=0.

Sekarang definisikan barisan suku-banyak sebagai berikut:

P_0(z)=1 \qquad P_{1}(z)=z \qquad  P_2(z)=z(z-1) \qquad  P_3(z)=z(z-1)(z-2) \cdots

Secara umum definisinya adalah sebagai berikut

P_k(z)=z(z-1)\cdots (z-k+1)

Untuk "turunan"-nya , kita akan menggunakan finite difference dari fungsi f, yakni
\Delta f(z) = f(z+1)-f(z)

Perhatikan bahwa operator \Delta (g(z)) bersifat linear, yakni \Delta(C  \cdot g(z)) = C \cdot \Delta (g(z)) dan \Delta (g_1(z)+g_2(z)) = \Delta (g_1(z)) + \Delta (g_2(z)).


Dengan melakukan finite difference pada suku-banyak P_k(z), kita  dapat memperoleh:

\Delta P_k(z) =  k P_{k-1}(z).

Relasi diatas cukup similar dengan rumus turunan \frac{d}{dz}(M_k(z)) = k M_{k-1}(z).

Dengan bekal properti diatas,  kita akan mencoba untuk mendapatkan sebuah  expansi dari suatu fungsi f, yang "analog" dengan  expansi Maclaurin-Taylor. (bahkan penurunannya juga  mirip.)

Expansi berikut disebut  Newton Series:

f(z)=\sum_{k=0}^{\infty} a_k P_k(z) = a_0 + a_1 z +a_2 (z)(z-1) + a_3 z(z-1)(z-2)+ \cdots

Perhatikan bahwa

\begin{align*}\Delta^2 f(z) &= \Delta ( \Delta f(z) ) \\  &= \Delta ( f(z+1)-f(z)) = (f(z+2)-f(z+1)) - (f(z+1)-f(z)) \\ &= f(z+2) -2f(z+1)+f(z) \end{align*}


\begin{align*}\Delta^3 f(z)  &= \Delta (f(z+2) -2f(z+1)-f(z))\\ &= f(z+3)-2f(z+2)+f(z+1) - f(z+2)+2f(z+1)-f(z) \\ &=f(z+3)-2f(z+2)+3f(z+1)-f(z) \end{align*}

Pola koefisien diatas menurut pada koefisien binomial, dan hal ini  cukup mudah untuk  dibuktikan dengan induksi, yakni

\Delta^n (f(z)) = \sum_{k=0}^n (-1)^{n-k}{n \choose k}f(z+k)

Untuk z=0, kita peroleh

\begin{equation}\Delta^n (f(0)) =\sum_{k=0}^n(-1)^{n-k} {n\choose k} f(k)  \qquad   \qquad (1) \end{equation}

Sekarang kembali lagi ke  Newton Series f(z)=\sum_{k=0}^n a_k P_k(z).  Perhatikan bahwa karena P_k(0)=0 untuk k\geq 1, maka f(0)=a_0.  Lalu dengan melakukan finite difference ke fungsi  f(z), diperoleh

\Delta (f(z)) = \sum_{k=0}^{\infty}  a_k \Delta P_k(z)  = \sum_{k=0}^n  a_k \cdot k \cdot P_{k-1}(z) = \sum_{k=1}^{\infty} k  a_ k  P_{k-1}(z)

Sehingga \Delta (f(0)) = a_1

Lebih lanjut diperoleh

\Delta^2 (f(z)) = \sum_{k=1}^{\infty} ka_k (k-1) P_{k-2}(z) = \sum_{k=2}^{\infty} ka_k (k-1) P_{k-2}(z).

Sehingga \Delta^2 (f(0)) = 2 a_2

Dengan menggunakan induksi dan cara diatas, dapat diperoleh bahwa \Delta^n(f(0)) = n!  a_n, sehingga

f(z)= \sum_{n=0}^{\infty}  \frac{\Delta^n(f(0))}{n!} P_n(z)

jadi
f(z)=\sum_{n=0}^{\infty}  \frac{\Delta^n(f(0))}{n!} z(z-1)  \cdots  (z-n+1)    \qquad \qquad (2)

voilà!

Untuk kasus khusus  ketika f(z)=c_m z^m + c_{m-1} z^{m-1} + \cdots +c_0, yang  merupakan  suku-banyak dengan derajat m, maka suku-suku pada expansi  diatas harus  berhenti, yakni pada pangkat tertinggi  dari f(z), sehingga diperoleh

f(z)= \sum_{n=0}^m \frac{\Delta^n(f(0))}{n!} z(z-1)  \cdots  (z-k+1)  

Dengan menyamakan  koefisien  dan  persamaan (1),  kita peroleh

m! c_m = \Delta^m (f(0)) = \sum_{k=0}^m (-1)^{m-k} {m \choose k} f(k)

dimana c_m adalah koefisien tertinggi  dari suku-banyak f(z).

Perlu diketahui juga bahwa himpunan suku-banyak \{P_k(X)\}_{k \geq 0} merupakan himpunan yang bebas linear di \mathbb{Q}.  Karena jika

\alpha_1 P_{k_1}(X)+\cdots + \alpha_m P_{k_m}(X)=0

dimana asumsikan k_1 > k_2 >  \cdots > k_n dan \alpha_i \in \mathbb{Q}, kita peroleh dengan substitusi t=k_n, maka diperoleh P_{k_{j}}(t) = 0 untuk j < n,  dan P_{k_n}(k_n)= (k_n)! \neq 0, sehingga

\alpha_n (k_n)!  = 0 \Rightarrow \alpha_n =0

apabila dilanjutkan dengan t=k_{n-1}, dan seterusnya, maka diperoleh semua \alpha_i = 0. Jadi \{P_k(X)\}_{k \geq 0} bebas linear.



Expansi diatas juga dapat memberikan syarat cukup dan perlu untuk sebuat suku banyak P(X) dengan koefisien rational yang memenuhi P(n) \in \mathbb{Z} untuk setiap n \in \mathbb{Z}, atau biasa disebut integer-valued polynomials.  Contoh P(n)=\frac{n(n+1)}{2}.

Sebuah  suku-banyak P(X) \in \mathbb{Q}[X] dimana deg(P(x))=n merupakan suku-banyak integer-valued  jika dan hanya jika \Delta^k P(x) \in \mathbb{Z} untuk setiap bilangan bulat k=0,2,..,n.  Jika dan hanya jika P(X) = \sum_{j=0}^n a_j {X \choose j} \quad \qquad \text{dimana }a_j \in \mathbb{Z}

Buktinya cukup mudah, Jika P(x) integer valued, maka dari rumus \Delta^n (P(0)) =\sum_{k=0}^n(-1)^{n-k} {n\choose k} P(k), kita peroleh \Delta^n (P(0)) selalu bilangan bulat. Sebaliknya jika \Delta^k P(x) \in \mathbb{Z} untuk setiap bilangan bulat k=0,2,..,n, maka karena {X \choose j} selalu bilangan bulat untuk setiap j, diperoleh  P(X) = \sum_{j=0}^n a_j {X \choose j} \quad \qquad \text{dimana }a_j \in \mathbb{Z}, dengan a_j = \Delta^j (P(0)).




Berikut ini adalah salah satu penggunaan identitas (2) pada soal USA TST 2019

Misalkan \mathbb{Z}/n\mathbb{Z} menyatakan himpunan bilangan  bulat dalam modulo n (jadi mempunyai n  anggota). Tentukan semua bilangan asli n sedemikian sehingga terdapat fungsi bijektif g: \mathbb{Z}/n \mathbb{z} \rightarrow \mathbb{Z}/n \mathbb{Z} sedemikians sehingga 101 buah fungsi
g(x) , \quad  g(x)+x , \quad g(x)+2x ,  \quad , \cdots, \quad ,  g(x)+100x
juga merupakan fungsi bijektif.

Solusi berikut  merupakan solusi dari Evan Chen.  Tapi soal ini sebenarnya tidak begitu susah apabila kita sudah mengetahui rumus penjumlahan \sum_{k=1}^m k^j,  yang melibatkan bilangan  Stirling, dan juga dapat diturunkan dari rumus Newton Series di atas.

Intinya, seperti biasa , dalam mengerjakan soal, kita coba lihat untuk kasus kecil dulu, yakni ketika l=101 diganti, misalkan l=2 atau l=3.

Pada kasus l=2 kita ingin fungsi g(x) dan g(x)+x merupakan fungsi bijektif, atau permutasi dari (1,2,...,n). Jadi apabila kita tambahkan diperoleh

\sum_{k=1}^n g(k) = \frac{n(n+1)}{2} \qquad \sum_{k=1}^n  g(x)+x = \frac{n(n+1)}{2}

sehingga diperoleh \frac{n(n+1)}{2} + \frac{n(n-1)}{2} \equiv \frac{n(n+1)}{2} \pmod n.

jadi  \frac{n(n+1)}{2} \equiv 0 \pmod n, sehingga n bilangan ganjil.

Untuk tiga buah fungsi  g(x),  g(x)+x, dan g(x)+2x kita bisa buat menjadi

g(x)^2  - 2(g(x)+x)^2 + (g(x)+2x)^2 = 2x^2

Sehingga

\begin{align*}\sum_{k=1}^n 2k^2 &= \sum_{k=1}^n g(k)^2 - 2 \sum_{k=1}^n (g(k)+2k)^2 + \sum_{k=1}^n (g(k)+2k)^2\\ &= \sum_{k=1}^nk^2 - 2  \sum_{k=1}^n k^2  + \sum_{k=1}^n k^2  \\ &=0  \end{align*}

Jadi \frac{(n)(n+1)(2n+1)}{3} \equiv 0  \pmod n, sehingga disimpulkan  3\nmid n.

Sebenarnya dapat dilihat bahwa pola yang dibutuhkan adalah membentuk g(x), g(x)+x, g(x)+2x, ...,  g(x)+kx menjadi k!x^k.   Dan pola ini berdasarkan dua kasus kecil diatas, seperti baris pada segitiga pascal (ganti tanda) yakni:  (1,-1) dan (1,-2,1).

Untuk membuktikan pola ini, kita dapat menggunakan rumus (2) diatas pada fungsi w(k)=x\cdot k + g(x), yaitu merupakan fungsi linear dalam variabel k. Pertatikan  bahwa  w(k)^m  merupakan suku-banyak dalam k dan berderajat m, dengan koefisien tertinggi sama-dengan  x^m. Jadi  apabila kita pakai rumus  diatas diperoleh

m! x^m = \Delta^m (w(0)^m) = \sum_{k=0}^m (-1)^{m-k} {m \choose k} w(k)^m  =    \sum_{k=0}^m (-1)^{m-k} {m \choose k}  (g(x)+k\cdot x)^m

Jadi

m! \sum_{x \in \mathbb{Z}/n\mathbb{Z}} x^m  \equiv \sum_{k=0}^m  (-1)^{m-k} {m \choose k} \left( \sum_{x \in \mathbb{Z}/n\mathbb{Z}}(g(x)+k\cdot x)^m\right)  

Sedangkan dari hipotesa  (g(x)+k\cdot x)^m bijektif kita peroleh

\sum_{x \in \mathbb{Z}/n\mathbb{Z}}(g(x)+k\cdot x)^m  \equiv  0^m + 1^m + \cdots+ (n-1)^m \equiv l_{n-1}

untuk setiap k.

Jadi diperoleh

m! \sum_{x \in \mathbb{Z}/n\mathbb{Z}} x^m \equiv l_{n-1}\left( \sum_{k=0}^m (-1)^{m-k}{m \choose k}\right) \equiv l_n \cdot 0=  0 \pmod n

Relasi diatas berlaku untuk m=1,2,..., 100, dapat diperiksa bahwa untuk k=0, rumus juga berlaku.

Kita akan buktikan bahwa hal  ini menyebabkan p \nmid n untuk setiap bilangan prima p \leq 101.

Jika ada bilangan prima 3 \leq p \leq  101 (karena p=2 sudah diperoleh diatas) sehingga p  \lvert n, asumsikan p prima terkecil yang demikian, dan v_p(n)=a.  Dengan menggunakan m=p-1, diperoleh

(p-1)! \sum_{x \in \mathbb{Z}/n\mathbb{Z}} x^{p-1} \equiv 0 \pmod n

sehingga dari (p-1)! \neq  0 \pmod {p^a}  dan p^a \lvert n diperoleh:

p^a \big \lvert  \sum_{x \in \mathbb{Z}/n\mathbb{Z}} x^{p-1}

Misalkan S_p \equiv  \sum_{x=1}^{p^a} x^{p-1} \pmod {p^a} , maka

\sum_{k=1}^{p^a} (p^a+k)^{p-1} \equiv \sum_{k=1}^{p^a} (2p^a+k)^{p-1} \equiv \cdots \equiv \sum_{k=1}^{p^a} ((n/p^a-1) p^a+k)^{p-1} \equiv S_p \pmod {p^a}

Jadi

0 \equiv \sum_{x \in  \mathbb{Z}/n\mathbb{Z}} x^{p-1} \equiv \frac{n}{p^a} \times S_p \pmod {p^{a}}

dan karena p^a \nmid  \frac{n}{p^a}, diperoleh

p^a \big\lvert \sum_{k=1}^{p^a} k^{p-1}

Kita akan buktikan ini tidak mungkin untuk a \geq 1, yakni dengan menggunakan   induksi matematika, kita akan buktikan bahwa v_{p}\left( \sum_{k=1}^{p^a} k^{p-1}\right)=a-1.

Jika a=1, maka diperoleh k^{p-1} \equiv 1 \pmod  p, untuk k \neq p, sehingga

\sum_{k=1}^{p} k^{p-1} \equiv p-1 \not \equiv 0 \pmod p.
jadi v_p\left(\sum_{k=1}^{p} k^{p-1}\right)=0.

Sekarang misalkan benar untuk suatu a \geq  1, kita bisa tulis

\sum_{k=1}^{p^a} k^{p-1} = \sum_{(k,p^a)  = 1} k^{p-1} + \sum_{k=1}^{p^{a-1}} (k \cdot p)^{p-1}


Sekarang misalkan g adalah primitive  root dari \mathbb{Z}/p^a\mathbb{Z}, kita peroleh \{x \, : (x,p^a)=1\} = \{1,g, g^2, ...,  g^{p^{a-1}(p-1)-1}\}. Sehingga

\sum_{(k,p^a)  = 1} k^{p-1} \equiv  1+g^{p-1} + g^{2(p-1)}  + \cdots +g^{(p^a(p-1)-1)(p-1)} \equiv  \frac{(g^{p-1})^{p^{a-1}(p-1)}-1 }{g^{p-1}-1}  \pmod {p^a}

Sedangkan karena p  \lvert g^{p-1}-1, dengan Lifting The  Exponent diperoleh

v_p((g^{p-1})^{p^{a-1}(p-1)}-1)  = v_p(g^{p-1}-1 ) + v_p(p^{a-1}(p-1)) =v_p(g^{p-1}-1 )  + a-1  

Jadi

v_p \left(\frac{(g^{p-1})^{p^{a-1}(p-1)}-1 }{g^{p-1}-1}  \right)  =  a-1 < a

sehingga
\sum_{(k,p^a)  = 1} k^{p-1} =  d_1 \cdot p^{a-1}
untuk suatu  bilangan d_1 yang relatif prima dengan p.

Jadi dengan hipotesa induksi

\begin{align*}\sum_{k=1}^{p^a} k^{p-1} &= p^{a-1} d_1 + p^{p-1}\left( \sum_{k=1}^{p^{a-1}}  k^{p-1} \right) \\  &=  p^{a-1} d_1 + p^{p-1} \left(d_2 p^{a-2} \right)  \\&= p^{a-1} (d_1 + p^{p-2} d_2)\end{align*}

karena p \nmid d_1 + p^{p-2} d_2, induksi selesai.

Dengan demikian dapat disimpulkan bahwa tidak ada bilangan prima p \leq 101  yang memenuhi p \lvert n.

Jadi diperoleh semua bilangan prima yang \leq  101 harus tidak membagi n. Untuk  contoh fungsi yang memenuhi ini, cukup ambil g(x)=x. Maka g(x)+kx harus bijective untuk 1 \leq k \leq 100. Karena jika x  dan y memenuhi g(x)+kx \equiv g(y)+ky \pmod n, maka diperoleh (x-y)(k+1) \equiv 0 \pmod n,   yang karena  (k+1,n)=1, diperoleh x \equiv y  \pmod n. Selesai.


Soal berikut ini (yang juga ada pada pelatihan) dapat dikerjakan dengan menggunakan syarat cukup dan perlu untuk suku-banyak bernilai bilangan bulat.

Tunjukkan  bahwa barisan (\epsilon_n)_{n \in\mathbb{N}}  yang bernilai plus-minus satu bersifat periodik dengan periode bilangan 2 berpangkat, jika dan hanya jika \epsilon_n = (-1)^{P(n)} dimana P(n) adalah suku-banyak dengan koefisien bilangan rasional dan bernilai bilangan bulat untuk setiap bilangan bulat.







Post a Comment: